1 Introduction

To study the boundedness of solutions for some nonautonomous second order linear differential equations, Ou-Iang [1] used a nonlinear integral inequality. This type of integral inequality had been also used to obtain global existence, uniqueness and stability properties of various nonlinear differential equations. Pachpatte [2] gave the generalized Ou-Iang type integral inequality. In 1996, Constantin [3] established the following interesting alternative result for this generalization.

Theorem 1.1

If for some \(k,T>0\), \(u\in C(\mathbb{R}_{0}^{+},\mathbb{R}_{0}^{+})\) satisfies

$$u^{2}(t)\leq k^{2}+2\int_{0}^{t} \biggl\{ f(s)u(s) \biggl[u(s)+\int_{0}^{s} g(\tau )w\bigl(u(\tau)\bigr)\,d\tau \biggr]+h(s)u(s) \biggr\} \,ds, $$

\(\forall t\in[0,T]\), where \(f,g,h \in C(\mathbb{R}_{0}^{+},\mathbb{R}_{0}^{+})\) and w belongs to the class of continuous nondecreasing functions on \(\mathbb{R}_{0}^{+}\) such that \(w(r)>0\) if \(r>0\) and satisfies \(\int_{1}^{\infty} \frac{ds}{w(s)}=\infty\), then

$$u(t)\leq K(t)+\int_{0}^{t} f(s)G^{-1} \biggl\{ G\bigl(K(t)\bigr)+\int_{0}^{s} \bigl[f( \tau)+g(\tau )\bigr]\,d\tau \biggr\} \,ds, $$

where \(K(t)=k+\int_{0}^{t} h(s)\,ds\), \(G(r)=\int_{1}^{r} \frac{ds}{s+w(s)}\), \(r>1\). \(G^{-1}\) denotes the inverse function of G.

Applying above Theorem 1.1 and a topological transversality theorem, he showed that, under some suitable assumptions, the integrodifferential equation

$$ x^{\prime}(t)=F \biggl(t,x(t),\int_{0}^{t} K \bigl[t,s,x(s) \bigr]\,ds \biggr) $$

has a solution and gave bounds on that solution. Additionally, Yang and Tan [4] gave the generalization of Constantin’s inequality, and they also presented the discrete analogue of this inequality which is stated as follows.

Theorem 1.2

Let the function \(w\in C( \mathbb{R}_{+}, \mathbb{R}_{+})\) be nondecreasing, \(w(r)>0 \) for \(r>0\), \(\phi\in C^{1}( \mathbb{R}_{+}, \mathbb{R}_{+})\) with \(\phi'\) being nonnegative and nondecreasing. \(u,c \in C( \mathbb{N}_{M}, \mathbb{R}_{+})\) with \(c(n)\) nondecreasing. Further, let

$$f(n,s),g(n,s),h(n,s)\in C( \mathbb{N}_{M} \times \mathbb{N}_{M}, \mathbb {R}_{+}) $$

be nondecreasing with respect to n for every s fixed. Then the discrete inequality

$$\begin{aligned} \phi\bigl[u(n)\bigr] \leq& c(n)+\sum_{s=0}^{n-1} \Biggl\{ f(n,s) \phi'\bigl[u(s)\bigr] \times \Biggl[u(s)+\sum _{\xi=0}^{s-1} g(s,\xi)w\bigl(u(\xi)\bigr) \Biggr] \\ &{}+h(n,s) \phi'\bigl[u(s)\bigr]\Biggr\} ,\quad n\in \mathbb{N}_{M} \end{aligned}$$

implies

$$u(n)\leq L(n)+\sum_{s=0}^{n-1}f(n,s) G^{-1} \Biggl( G\bigl[L(n)\bigr]+ \sum_{\xi =0}^{s-1}f(n, \xi)+g(n,\xi) \Biggr). $$

Here \(G(r)=\int_{r_{0}}^{r} \frac{ds}{s+w(s)}\), \(r\leq r_{0}\), \(1>r_{0}>0\), \(\lim_{x\rightarrow\infty}G(x)=\infty\), \(L(n)=\phi^{-1}[c(n)]+\sum_{s=0}^{n-1} h(n,s)\), \(\mathbb{N}_{M}=\{n\in\mathbb{N}: n\leq M , M \in\mathbb{N}\}\).

2 Some basic definitions related to time scales

Using [57] we give the following information. By a time scale, denoted by \(\mathbb{T}\), we mean a nonempty closed subset of \(\mathbb{R}\). The theory of time scales gives a way to unify continuous and discrete analysis.

The set \(\mathbb{T}^{\kappa}\) is defined by \(\mathbb{T}^{\kappa}=\mathbb{T}/ (\rho(\sup\mathbb{T}),\sup\mathbb{T}]\) and the set \(\mathbb{T}_{\kappa}\) is defined by \(\mathbb{T}_{\kappa}=\mathbb{T} / [\inf\mathbb{T},\sigma(\inf \mathbb{T}))\). The forward jump operator \(\sigma:\mathbb{T}\rightarrow \mathbb{T}\) is defined by \(\sigma(t):=\inf(t,\infty)_{\mathbb{T}}\), for \(t\in \mathbb{T}\). The backward jump operator \(\rho:\mathbb{T}\rightarrow \mathbb{T} \) is defined by \(\rho(t):=\sup(-\infty,t)_{\mathbb{T}}\), for \(t\in\mathbb{T}\). The forward graininess function \(\mu:\mathbb{T}\rightarrow\mathbb{R}_{0}^{+}\) is defined by \(\mu(t):=\sigma(t)-t\), for \(t\in\mathbb{T}\). The backward graininess function \(\nu:\mathbb{T}\rightarrow\mathbb{R}_{0}^{+}\) is defined by \(\nu(t):=t-\rho(t)\), for \(t\in\mathbb{T}\). Here it is assumed that \(\inf\emptyset= \sup\mathbb{T}\) and \(\sup\emptyset=\inf\mathbb{T}\).

For a function \(f:\mathbb{T} \rightarrow\mathbb{T}\), we define the Δ-derivative of f at \(t\in\mathbb{T}^{\kappa}\), denoted by \(f^{\Delta}(t)\) for all \(\epsilon>0\). There exists a neighborhood \(U\subset\mathbb{T}\) of \(t\in\mathbb{T}^{\kappa}\) such that

$$ \bigl\vert f\bigl(\sigma(t)\bigr)-f(s)-f^{\Delta}(t) \bigl(\sigma(t)-s \bigr)\bigr\vert \leq\epsilon\bigl\vert \sigma(t)-s\bigr\vert $$

for all \(s\in U\).

For the same function define the ∇-derivative of f at \(t\in \mathbb{T}_{\kappa}\), denoted by \(f^{\nabla}(t)\), for all \(\epsilon>0\). There exists a neighborhood \(V\subset\mathbb{T}\) of \(t\in\mathbb{T}_{\kappa}\) such that

$$ \bigl\vert f(s)-f\bigl(\rho(t)\bigr)-f^{\nabla}(t) \bigl(s-\rho(t)\bigr) \bigr\vert \leq\epsilon\bigl\vert s-\rho(t)\bigr\vert $$

for all \(s\in V\).

We define \({\diamond_{\alpha}}\)-derivative of f at \(t\in\mathbb{T}^{\kappa}_{\kappa}\), denoted by \(f^{\diamond_{\alpha}}(t)\) for all \(\epsilon>0\). There is a neighborhood \(U\subset\mathbb{T}\) such that for any \(s\in U\),

$$\begin{aligned}& \bigl\vert \alpha\bigl\vert f\bigl(\sigma(t)\bigr)-f(s)\bigr\vert \bigl\vert \rho(t)-s\bigr\vert +(1-\alpha)\bigl\vert f\bigl(\rho(t)\bigr)-f(s) \bigr\vert \bigl\vert \sigma(t)-s\bigr\vert -f^{\diamond_{\alpha}}(t) \bigl\vert \rho(t)-s\bigr\vert \bigl\vert \sigma(t)-s\bigr\vert \bigr\vert \\& \quad \leq \epsilon\bigl\vert \rho(t)-s\bigr\vert \bigl\vert \sigma(t)-s \bigr\vert . \end{aligned}$$

A function \(f:\mathbb{T} \rightarrow\mathbb{R}\) is rd-continuous if it is continuous at right-dense points in \(\mathbb{T}\) and its left-sided limits exist at left-dense points in \(\mathbb{T}\). The class of real rd-continuous functions defined on a time scale \(\mathbb{T}\) is denoted by \(C_{\mathrm{rd}}(\mathbb{T},\mathbb{R})\). If \(f\in C_{\mathrm{rd}}(\mathbb{T},\mathbb{R})\), then there exists a function \(F(t)\) such that \(F^{\Delta }(t) = f (t)\). The delta integral is defined by \(\int_{a}^{b} f(x) \Delta x=F(b)-F(a)\).

Similarly, a function \(g:\mathbb{T} \rightarrow\mathbb {R}\) is ld-continuous if it is continuous at left-dense points in \(\mathbb{T}\) and its right-sided limits exist at right-dense points in \(\mathbb{T}\). The class of real ld-continuous functions defined on a time scale \(\mathbb{T}\) is denoted by \(C_{\mathrm{ld}}(\mathbb{T},\mathbb{R})\). If \(g\in C_{\mathrm{ld}}(\mathbb {T},\mathbb{R})\), then there exists a function \(G(t)\) such that \(G^{\nabla }(t) =g (t)\). The nabla integral is defined by \(\int_{a}^{b} g(x) \nabla x=G(b)-G(a)\).

By [8], if a function \(h(t):\mathbb{T} \rightarrow \mathbb{R}\) is continuous, then it is diamond-alpha integrable, and the fundamental theorem of calculus is not true for \({\diamond_{\alpha }}\)-derivative. We know that

$$ \biggl(\int_{a}^{t} h(s)\diamond_{\alpha}s \biggr)^{\diamond_{\alpha }}=\bigl(1-2\alpha+2 {\alpha}^{2}\bigr) h(t)+\bigl( \alpha-\alpha^{2}\bigr)\bigl[h\bigl(\rho(t)\bigr)+h\bigl(\sigma(t)\bigr) \bigr]. $$

Ferreira [9] generalized Constantin’s inequality involving delta derivatives on an arbitrary time scale.

Theorem 2.1

Assume that \(u\in C_{\mathrm{rd}} ( [ a,b ] _{ \mathbb{T}}, \mathbb{R}_{0}^{+} ) \), \(c\in C_{\mathrm{rd}} ( [ a,b ] _{ \mathbb{T}} , \mathbb{R} ^{+} ) \) is nondecreasing, \(\Phi\in C ( \mathbb{R}_{0}^{+}, \mathbb{R}_{0}^{+} ) \) is a strictly increasing function such that

$$\lim_{x\rightarrow\infty}\Phi(x)=\infty. $$

Let \(f(t,\xi),h(t,\xi)\in C_{\mathrm{rd}} ( [ a,b ] _{ \mathbb{T}}\times [ a,b ] _{ \mathbb{T}^{\kappa}}, \mathbb{R} _{0}^{+} ) \) and \(g(t,\xi)\in C_{\mathrm{rd}} ( [ a,b ] _{ \mathbb{T}^{\kappa}}\times [ a,b ] _{ \mathbb{T}^{{\kappa }^{2}}}, \mathbb{R}_{0}^{+} ) \) be nondecreasing for every fixed ξ. Further, let \(w,\phi,\Psi\in C ( \mathbb {R}_{0}^{+}, \mathbb{R}_{0}^{+} ) \) be nondecreasing such that \(\{ w,\phi,\Psi \} (x)>0\) for every \(x>0\). Define

$$M(x)=\max \bigl\{ \phi(x),\Psi(x) \bigr\} $$

on \(\mathbb{R} _{0}^{+}\) and assume that the following function

$$F(x)=\int_{x_{0}}^{x}\frac{ds}{M\circ \Phi^{-1}(s)} $$

with \(x>c(a)>x_{0}\geq0\) if \(\int_{0}^{x}\frac{ds}{M\circ \Phi ^{-1}(s)}<\infty\) and \(x>c(a)>x_{0}>0\) if \(\int_{0}^{x}\frac{ds}{M\circ \Phi ^{-1}(s)}=\infty\) satisfies \(\lim_{x\rightarrow\infty}F(x)=\infty\).

Also assume that the function

$$P(r)=\int_{r_{0}}^{r}\frac{ds}{w(s)}, $$

where \(r\geq0\), \(r_{0}\geq0\) if \(\int_{0}^{r}\frac {ds}{w(s)}<\infty\) and \(r>0\), \(r_{0}>0\) if \(\int_{0}^{r}\frac {ds}{w(s)}=\infty\) satisfies \(\lim_{r\rightarrow\infty}P(r)=\infty\).

If \(\Phi^{-1} [ F^{-1}(x) ] \leq x\), for all \(x\geq0\), the inequality

$$\Phi \bigl[ u(t) \bigr] \leq c(t)+\int_{a}^{t} \biggl( f(t,s)\phi \bigl[ u(s) \bigr] \biggl[ u(s)+\int_{a}^{s}g(s, \xi)w \bigl[ u(s) \bigr] \Delta\xi \biggr]+h(t,s)\Psi \bigl[ u(s) \bigr] \biggr) \Delta s $$

for \(t\in [ a,b ] _{T}\) implies

$$u(t)\leq\Phi^{-1} \biggl[ F^{-1} \biggl( K(t)+ \int _{a}^{t} \biggl( f(t,s)G^{-1} \biggl[ G \bigl(K(t)\bigr)+ \int_{a}^{s}\max \bigl\{ f(t, \xi),g(t,\xi) \bigr\} \Delta\xi \biggr] \biggr) \Delta s \biggr) \biggr]. $$

Here \(K(t)=:F [ c(t) ] + \int_{a}^{t}h(t,s)\Delta s \) and \(G(r)=\int_{r_{0}}^{r}\frac{ds}{w(s)+s}\).

3 Some new results

We try to generalize Constantin’s inequality containing nabla and diamond-alpha derivatives and present the results we have obtained.

First we look for the discrete analogue of Constantin’s inequality involving nabla derivatives.

Theorem 3.1

Let the function \(w\in C( \mathbb{R}_{+}, \mathbb{R}_{+})\) be nondecreasing, \(w(r)>0 \) for \(r>0\), \(\phi\in C^{1}( \mathbb{R}_{+}, \mathbb{R}_{+})\) with \(\phi'\) being nonnegative and nondecreasing. \(u,c \in C( \mathbb{N}_{M}, \mathbb{R}_{+})\) with \(c(n)\) nondecreasing. Further, let

$$f(n,s),g(n,s),h(n,s)\in C( \mathbb{N}_{M} \times \mathbb{N}_{M}, \mathbb {R}_{+}) $$

be nondecreasing with respect to n for every s fixed. Then there exist fixed constants \(k,l>0\) such that the discrete inequality

$$\begin{aligned} \phi\bigl[u(n)\bigr] \leq& c(n)+\sum_{s=1}^{n} \Biggl\{ f(n,s) \phi'\bigl[u(s)\bigr] \times \Biggl[u(s)+\sum _{\xi=1}^{s} g(s,\xi)w\bigl(u(\xi)\bigr) \Biggr] \\ &{}+h(n,s) \phi'\bigl[u(s)\bigr]\Biggr\} , \quad n\in \mathbb{N}_{M} \end{aligned}$$

implies

$$u(n)\leq L(n)+\sum_{\xi=1}^{n}kf(n,\xi) G^{-1} \Biggl( G\bigl[L(n)\bigr]+ l\sum_{s=1}^{\xi}kf(n,s)+g(n,s) \Biggr). $$

Here \(G(r)=\int_{r_{0}}^{r} \frac{ds}{s+w(s)}\), \(r\leq r_{0}\), \(1>r_{0}>0\), \(\lim_{x\rightarrow\infty}G(x)=\infty\), \(L(n)=\phi^{-1}[c(n)]+k\sum_{s=1}^{n} h(n,s)\), \(\mathbb{N}_{M}=\{n\in N: n\leq M , M \in\mathbb{N}\}\). Assume that \(\mathbb{N}\) starts with 1.

Proof

Fixing an arbitrary positive integer \(m\in[1,M]\).

We denote the set \(J=\{1,2,\ldots,m\}\) and we define a positive function \(z(n)\in K(J, \mathbb{R}_{+})\) such that

$$ z(n)=c(m)+\sum_{s=1}^{n} \Biggl\{ f(m,s)\phi'\bigl[u(s)\bigr] \Biggl[u(s)+\sum _{\xi=1}^{s} g(m,\xi)w\bigl(u(\xi)\bigr) \Biggr]+h(m,s) \phi'\bigl[u(s)\bigr] \Biggr\} . $$
(1)

Then \(u(n)\leq\phi^{-1}[z(n)]\). Using (1) we get

$$ \frac{\nabla z(n)}{\phi'[\phi^{-1}[z(n)]]}\leq f(m,n)\Biggl[\phi^{-1}\bigl[z(n) \bigr]+\sum_{s=1}^{n} g(m,s)w\bigl( \phi^{-1}\bigl[z(n)\bigr]\bigr)\Biggr]+h(m,n). $$
(2)

Since we have finitely many elements in the domain, functions are bounded and, since each of them goes to \(\mathbb{R}_{+}\), it never takes 0. Then

$$\begin{aligned} k =&\max \biggl\{ \frac{\phi'[\phi^{-1}[z(m)]]}{\phi'[\phi^{-1}[z(m-1)]]},\frac {\phi'[\phi^{-1}[z(m-1)]]}{\phi'[\phi^{-1}[z(m-2)]]},\ldots, \\ &\frac{\phi '[\phi^{-1}[z(n)]]}{\phi'[\phi^{-1}[z(n-1)]]}, \ldots,\frac{\phi'[\phi ^{-1}[z(1)]]}{\phi'[\phi^{-1}[z(0)]]} \biggr\} \end{aligned}$$

exists.

If we multiply (2) by k and use the mean value theorem, we get

$$\nabla\phi^{-1}\bigl[z(n)\bigr]\leq kf(m,n) \Biggl[\phi^{-1} \bigl[z(n)\bigr]+\sum_{s=1}^{n} g(m,s)w\bigl( \phi^{-1}\bigl[z(n)\bigr]\bigr) \Biggr]+ kh(m,n). $$

Substituting n with ξ in the last assertion and summing over \(\xi =1,\ldots,n\), we have

$$\begin{aligned} \phi^{-1}\bigl[z(n)\bigr] \leq&\phi^{-1}\bigl[z(0)\bigr]+k\sum _{\xi=1}^{m} h(m,\xi) \\ &{}+k\sum _{\xi =1}^{n} f(m,\xi) \Biggl[\phi^{-1}\bigl[z( \xi)\bigr]+\sum_{s=1}^{\xi}g(m,s)w\bigl[\phi ^{-1}\bigl[z(s)\bigr]\bigr] \Biggr]. \end{aligned}$$

Define the right-hand side of the last inequality as \(v(n)\), then we get \(\phi^{-1}[z(n)]\leq v(n)\) for \(n\in J\). Taking the nabla derivative of \(v(n)\), we obtain

$$\nabla v(n)\leq kf(m,n) \Biggl[v(n)+\sum_{s=1}^{n}g(m,s)w \bigl[v(s)\bigr] \Biggr],\quad n\in J. $$

We define \(y(n)=v(n)+\sum_{s=1}^{n}g(m,s)w[v(s)]\) and take the nabla derivative of \(y(n)\), then we get

$$ \frac{\nabla y(n)}{y(n)+w(y(n))}\leq kf(m,n)+g(m,n). $$
(3)

For inequality (3), substituting n with s and summing over \(s=1,2,\ldots, n\), we obtain

$$ \sum_{s=1}^{n} \frac{\nabla y(s)}{y(s)+w(y(s))}\leq\sum_{s=1}^{n}kf(m,s)+g(m,s). $$
(4)

Let \(l=\max_{n\in[0,m]}\frac{\sum_{s=1}^{n}\frac{\nabla y(s)}{y(s-1)+w(y(s-1))}}{\sum_{s=1}^{n}\frac{\nabla y(s)}{y(s)+w(y(s))}}\). Multiply (4) by l and by the mean value theorem, we get

$$y(n)\leq G^{-1} \Biggl( G\bigl[y(0)\bigr]+ l\sum _{s=1}^{n}kf(m,s)+g(m,s) \Biggr). $$

Since \(\nabla v(n)\leq kf(m,n)y(n)\), we obtain

$$\nabla v(n)\leq kf(m,n) G^{-1} \Biggl( G\bigl[y(0)\bigr]+ l\sum _{s=1}^{n}kf(m,s)+g(m,s) \Biggr). $$

Substituting n with ξ and summing over \(\xi=1,2,\ldots,n\) and using \(u(n)\leq\phi^{-1}[z(n)]\leq v(n)\), we get

$$u(n)\leq L(m)+\sum_{\xi=1}^{n}kf(m,\xi) G^{-1} \Biggl( G\bigl[L(m)\bigr]+ l\sum_{s=1}^{\xi}kf(m,s)+g(m,s) \Biggr). $$

Hence we get the desired result. □

Example 3.1

Let

$$ u^{2}(n)=L+2\sum_{s=1}^{n}H(n-s)u(s). $$
(5)

Here \(n\in[1,M]\). The unique positive solution for equation (5) can be obtained by successive substitution. For instance, by letting \(n=1,2\) we obtain

$$\begin{aligned}& u(1)=\sqrt{L+2H(0)u(1)}, \\& u(2)=\sqrt{L+2H(1)u(1)+2H(0)u(2)}. \end{aligned}$$

By using solutions of quadratic equations, we can find \(u(0),u(1),u(2),\ldots ,u(M)\) successively. If we use the theorem above, then the bound for u will be \(u(n)\leq \sqrt{L}+ k\sum_{s=1}^{n}H(n-s)\). With the help of the proof of Theorem 3.1 here

$$ k=\max_{n\in[1,M]} \biggl[ 1+\frac{2H(M-n)u(n)}{L+2H(M-1)u(1)+2H(M-2)u(2)+\cdots+2H(M-n+1)u(n-1)} \biggr]. $$

Here \(f(n,s)=g(n,s)=0\), \(h(n,s)=H(n-s)\), \(c(n)=L>0\), L is constant, \(u,H\in K( \mathbb{N}_{M}, \mathbb{R}_{+})\), P is nondecreasing, and \(\phi (x)=x^{2}\).

If we choose \(H(n,s)\), L good enough, k does not exceed a number c when M tends to infinity. For instance, if \(H(n,s)=1\), \(L=3\), then k does not exceed 3 when M tends to infinity. Therefore, for the equality \(u^{2}(n)=3+2\sum_{s=1}^{n}u(s)\), by using the theorem above, we have the bound for

$$u(n)\leq\sqrt{3}+ 3n, \quad n\in\mathbb{N}. $$

Lemma 3.1

Let \(f: \mathbb{R}\rightarrow\mathbb{R}\) be continuously differentiable and \(g: \mathbb{T}\rightarrow\mathbb{R}\) be nabla differentiable, then \(f \circ g\) is nabla differentiable and the formula is given by

$$(f \circ g)^{\nabla}(t)=g^{\nabla}(t){ \biggl[\int_{0}^{1}f' \bigl(g\bigl(\rho(t)\bigr)+h\nu (t)g^{\nabla}(t)\bigr)\, dh \biggr]}. $$

Proof

Apply an ordinary substitution rule from calculus and get

$$f\bigl(g(s)\bigr)-f\bigl(g\bigl(\rho(t)\bigr)\bigr)=g(s)-g\bigl(\rho(t)\bigr) \int_{0}^{1}f^{\prime }\bigl(hg(s)-(1-h)g \bigl(\rho(t)\bigr)\bigr)\, dh. $$

Let \(t\in \mathbb{T}_{\kappa}\) and \(\epsilon>0\) be given. Since g is nabla differentiable at t, there exists a neighborhood \(U_{1}\) of t such that

$$\bigl\vert g(s)-g\bigl(\rho(t)\bigr)-g^{\nabla}(t) \bigl(s-\rho(t)\bigr) \bigr\vert \leq \epsilon^{\ast}\bigl\vert s-\rho(t)\bigr\vert ,\quad \forall s\in U_{1}, $$

where \(\epsilon^{\ast}=\frac{\epsilon}{1+2\int_{0}^{1} \vert f^{\prime}(hg(s)-(1-h)g(\rho(t)))\vert \, dh}\).

By assumption we have

$$\bigl\vert f^{\prime}\bigl(h\bigl(g(s)\bigr)+(1-h)g\bigl(\rho(t)\bigr) \bigr)-f^{\prime }\bigl(h\bigl(g(t)\bigr)+(1-h)g\bigl(\rho(t)\bigr)\bigr) \bigr\vert \leq\frac{\epsilon}{\epsilon ^{\ast }+\vert g^{\nabla}(t)\vert },\quad \forall s\in U_{2}, $$

where \(\forall\delta>0\) \(\exists U_{2}\) is a neighborhood of t such that \(\vert g(s)-g(t)\vert <\delta\), \(\forall s\in U_{2}\). Here let us define a neighborhood U of t such that \(U=U_{1}\cap U_{2}\). Then we have

$$\begin{aligned}& \biggl\vert f \circ g(s)-f \circ g\bigl(\rho(t)\bigr)-\bigl(s-\rho(t)\bigr)g^{\nabla }(t) \int_{0}^{1}f^{\prime}(\beta)\, dh\biggr\vert \\& \quad =\biggl\vert g(s)-g\bigl(\rho(t)\bigr)-\bigl(s-\rho(t) \bigr)g^{\nabla }(t)\int_{0}^{1}f^{\prime}( \alpha)\, dh+\bigl(s-\rho(t)\bigr)g^{\nabla }(t)\int_{0}^{1}f^{\prime}( \alpha)-f^{\prime}(\beta )\, dh\biggr\vert \\& \quad \leq\bigl\vert g(s)-g\bigl(\rho(t)\bigr)-\bigl(s-\rho(t) \bigr)g^{\nabla}(t)\bigr\vert \int_{0}^{1} \bigl\vert f^{\prime}(\alpha)\bigr\vert \, dh \\& \qquad {}+ \bigl\vert s-\rho(t) \bigr\vert \bigl\vert g^{\nabla}(t)\bigr\vert \int _{0}^{1}\bigl\vert f^{\prime}( \alpha)-f^{\prime}(\beta )\bigr\vert \, dh \\& \quad \leq\epsilon^{\ast}\bigl\vert s-\rho(t)\bigr\vert \int _{0}^{1}\bigl\vert f^{\prime}(\alpha)\bigr\vert \, dh+ \bigl\vert s-\rho(t)\bigr\vert \bigl\vert g^{\nabla}(t) \bigr\vert \int_{0}^{1}\bigl\vert f^{\prime}(\alpha)-f^{\prime}(\beta )\bigr\vert \, dh \\& \quad \leq\epsilon^{\ast}\bigl\vert s-\rho(t)\bigr\vert \int _{0}^{1}\bigl\vert f^{\prime}(\beta)\bigr\vert \, dh+ \bigl\vert s-\rho(t)\bigr\vert \bigl(\bigl\vert g^{\nabla}(t)\bigr\vert +\epsilon ^{\ast }\bigr)\int _{0}^{1}\bigl\vert f^{\prime}( \alpha)-f^{\prime}(\beta )\bigr\vert \, dh \\& \quad \leq\frac{\epsilon}{2} \bigl\vert s-\rho(t) \bigr\vert + \frac {\epsilon}{2}\bigl\vert s-\rho(t)\bigr\vert =\epsilon\bigl\vert s- \rho(t)\bigr\vert , \end{aligned}$$

where \(\alpha=hg(s)+(1-h)g(\rho(t))\) and \(\beta=hg(t)+(1-h)g(\rho(t))\). Hence \(f \circ g\) is nabla differentiable and its derivative is as claimed above. □

This lemma first occurred in the article of Atici et al. [10] as

$$ (f \circ g)^{\nabla}(t)=g^{\nabla}(t){ \biggl[\int_{0}^{1}f^{\prime}\bigl(g(t)+h\nu(t)g^{\nabla}(t)\bigr) \, dh \biggr]}. $$

With a counter example, we can show that their version of the formula is not true.

Example 3.2

Let \(g(x): \mathbb{Z}\rightarrow \mathbb{R}\) such that \(g(n)=\frac {1}{n}\), \(f(x): \mathbb{R}\rightarrow \mathbb{R}\) such that \(f(x)=x^{2}\). Therefore the first derivative of \(f(x)\) is continuous. If we apply the above formula to find the nabla derivative of the function \((f \circ g)(t)\), we get

$$\frac{1-2n}{(n^{2})(n-1)^{2}}= \biggl(\frac{1}{n^{2}} \biggr)^{\nabla}\neq \biggl({\frac{1}{n}} \biggr)^{\nabla}\int_{0}^{1} \biggl[2 \biggl(\frac{1}{n} \biggr)+2h \biggl(\frac{1}{n} \biggr)^{\nabla} \biggr]\, dh=\frac{3-2n}{(n^{2})(n-1)^{2}}. $$

Lemma 3.2

Let \(a,b\in \mathbb{T}\), consider the time scale \([ a,b ] _{ \mathbb{T}}\) and a function \(r\in C_{\mathrm{ld}}^{1} ( [ a,b ] _{ \mathbb{T}}, \mathbb{R} ) \) with \(r^{\nabla}(t)\geq 0\). Suppose that a function \(g\in C ( \mathbb{R} _{0}^{+}, \mathbb{R} _{0}^{+} )\) is positive and nondecreasing on \(\mathbb{R}\). Then, for each \(t\in [ a,b ] _{ \mathbb{T}}\), we have

$$G\bigl(r(t)\bigr)\leq G\bigl(r(a)\bigr)+\int_{a}^{t} \frac{r^{\nabla}(\tau )}{g(r(\rho(\tau)))}\nabla\tau, $$

where \(G(x)=\int_{x_{0}}^{x}\frac{ds}{g(s)}\), where \(x\geq0\), \(x_{0}\geq0\) if \(\int_{0}^{x}\frac{ds}{g(s)}<\infty\) and \(x>0\), \(x_{0}>0\) if \(\int_{0}^{x}\frac{ds}{g(s)}=\infty\).

Proof

Since g is positive and nondecreasing on \(( 0,\infty )\), we have

$$r^{\nabla}(t)\int_{0}^{1} \frac{dh}{g ( r(\rho (t))+h\nu(t)r^{\nabla}(t) ) } \leq\frac{r^{\nabla}(t)}{g ( r(\rho(t)) ) }. $$

By using Lemma 3.1, we get

$$( G\circ r ) ^{\nabla}(t)\leq\frac{r^{\nabla}(t)}{g ( r(\rho(t)) ) } $$

and

$$G\bigl(r(t)\bigr)\leq G\bigl(r(a)\bigr)+\int_{a}^{t} \frac{r^{\nabla}(\tau)}{ g ( r(\rho(\tau)) ) }\nabla\tau. $$

Hence we get the desired result. □

The delta derivative version of Lemma 3.1 was proved by Ferreira and Torres [11]. By using these results, we proved the following theorem.

Theorem 3.2

Assume that \(u\in C_{\mathrm{ld}} ( [ a,b ] _{ \mathbb{T}}, \mathbb{R}_{0}^{+} ) \), \(c\in C_{\mathrm{ld}} ( [ a,b ] _{ \mathbb{T}} , \mathbb{R} ^{+} ) \) is nondecreasing, \(\Phi\in C ( \mathbb{R}_{0}^{+}, \mathbb{R}_{0}^{+} ) \) is a strictly increasing function such that

$$\lim_{x\rightarrow\infty}\Phi(x)=\infty. $$

Let \(f(t,\xi),h(t,\xi)\in C_{\mathrm{ld}} ( [ a,b ] _{ \mathbb{T}}\times [ a,b ] _{ \mathbb{T}_{\kappa}}, \mathbb{R} _{0}^{+} ) \) and \(g(t,\xi)\in C_{\mathrm{ld}} ( [ a,b ] _{ \mathbb{T}_{\kappa}}\times [ a,b ] _{ \mathbb{T}_{{\kappa }^{2}}}, \mathbb{R}_{0}^{+} ) \) be nondecreasing in t for every fixed ξ. Further, let \(w,\phi,\Psi\in C ( \mathbb {R}_{0}^{+}, \mathbb{R}_{0}^{+} ) \) be nondecreasing such that \(\{ w,\phi,\Psi \} (x)>0\) for every \(x>0\). Assume that the following function

$$F(x)=\int_{x_{0}}^{x}\frac{ds}{M\circ \Phi^{-1}(s)} $$

with \(x>c(a)>x_{0}\geq0\) if \(\int_{0}^{x}\frac{ds}{M\circ \Phi ^{-1}(s)}<\infty\) and \(x>c(a)>x_{0}>0\) if \(\int_{0}^{x}\frac{ds}{M\circ \Phi ^{-1}(s)}=\infty\); \(\lim_{x\rightarrow\infty}F(x)=\infty\), where \(M(x)=\max \{ \phi(x),\Psi(x) \}\) on \(\mathbb{R} _{0}^{+}\).

Also assume that the function

$$P(r)=\int_{r_{0}}^{r}\frac{ds}{w(s)}, $$

where \(r\geq0\), \(r_{0}\geq0\) if \(\int_{0}^{r}\frac {ds}{w(s)}<\infty\) and \(r>0\), \(r_{0}>0\) if \(\int_{0}^{r}\frac {ds}{w(s)}=\infty\); \(\lim_{r\rightarrow\infty}P(r)=\infty\).

Then there exist fixed constants \(\alpha,\beta>0\) such that \(\Phi ^{-1} [ F^{-1}(x) ] \leq x\) for all \(x\geq0\), then

$$\Phi \bigl[ u(t) \bigr] \leq c(t)+\int_{a}^{t} \biggl( f(t,s)\phi \bigl[ u(s) \bigr] \biggl[ u(s)+\int_{a}^{s}g(s, \xi)w \bigl[ u(s) \bigr] \nabla\xi \biggr]+h(t,s)\Psi \bigl[ u(s) \bigr] \biggr) \nabla s $$

for \(t\in [ a,b ] _{\mathbb{T}}\) implies

$$\begin{aligned} u(t) \leq&\Phi^{-1} \biggl[ F^{-1} \biggl( K(t)+\alpha \int _{a}^{t} \biggl( f(t,s)G^{-1} \biggl[ G \bigl(K(t)\bigr) \\ &{}+\beta \int_{a}^{s}\max \bigl\{ \alpha f(t,\xi),g(t,\xi) \bigr\} \nabla\xi \biggr] \biggr) \nabla s \biggr) \biggr]. \end{aligned}$$

Here \(K(t):=F [ c(t) ] +\alpha\int_{a}^{t}h(t,s)\nabla s \) and \(G(r)=\int_{r_{0}}^{r}\frac{ds}{w(s)+s}\).

Proof

If \(t=a\), then, obviously, theorem holds. Let us fix an arbitrary number \(t_{0}\in \left.(a,b]\right._{ \mathbb{T}}\), we define \(z(t)\) on \([ a,t_{0} ] _{ \mathbb{T}}\) such that

$$z(t)=c(t_{0})+\int_{a}^{t} \biggl( f(t_{0},s)\phi\bigl(u(s)\bigr) \biggl[ u(s)+\int_{a}^{s}g(t_{0}, \xi)w\bigl(u(\xi)\bigr)\nabla\xi \biggr] +h(t_{0},s)\Psi \bigl[ u(s) \bigr] \biggr) \nabla s. $$

Since \(\Phi(u(t))\leq z(t)\), then \(u(t)\leq\Phi^{-1}(z(t))\), so we have

$$ \frac{z^{\nabla}(t)}{M [ \Phi ^{-1}(z(t)) ] }\leq f(t_{0},t) \biggl[ \Phi^{-1}\bigl(z(t)\bigr)+\int_{a}^{t}g(t_{0}, \xi)w \bigl[ \Phi ^{-1}\bigl(z(\xi)\bigr) \bigr] \nabla\xi \biggr] +h(t_{0},t). $$
(6)

Define \(\check{z}(t)\) on \([a,b]_{ \mathbb{T}}\) such that

$$\check{z}(t)=c(b)+\int_{a}^{t} \biggl( f(b,s) \phi \bigl[ u(s) \bigr] \biggl[ u(s)+\int_{a}^{s}g(b, \xi)w \bigl[ u(\xi) \bigr] \nabla \xi \biggr] +h(b,s)\Psi \bigl[ u(s) \bigr] \biggr) \nabla s. $$

It is obvious that \(z(t)\leq\check{z}(t)\) on \([ a,t_{0} ] _{ \mathbb{T}}\). Using \(\check{z}(t)\) we define α such that

$$\alpha=\max_{t\in [ a,t_{0} ] _{T}}\frac{M\circ \Phi^{-1}\circ \check {z}(t)}{M\circ \Phi^{-1}\circ z(a)}>0. $$

From the definition of the functions \(\check{z}(t)\), \(z(t)\), M, \(\Phi ^{-1}\) and by Theorem 1.65 from [12] α exists.

Multiply (6) by α and use Lemma 3.2 to get

$$\begin{aligned} z(t) \leq&F^{-1} \biggl[F\bigl(c(t_{0})\bigr)+\alpha\int _{a}^{t} \biggl( f(t_{0},s) \biggl[ \Phi^{-1}\bigl(z(s)\bigr)+\int_{a}^{s}g(t_{0}, \xi)w \bigl[ \Phi ^{-1}\bigl(z(\xi)\bigr) \bigr] \nabla\xi \biggr] \biggr) \nabla s \\ &{}+\alpha \int_{a}^{t_{0}}h(t_{0},s) \nabla s \biggr]= F^{-1}\bigl(y(t)\bigr), \end{aligned}$$

where

$$\begin{aligned} y(t) :=& F\bigl(c(t_{0})\bigr)+ \alpha\int_{a}^{t} \biggl( f(t_{0},s) \biggl[ \Phi ^{-1}\bigl(z(s)\bigr)+\int _{a}^{s}g(t_{0},\xi)w \bigl[ \Phi^{-1}\bigl(z(\xi)\bigr) \bigr] \nabla\xi \biggr] \biggr) \nabla s \\ &{}+\alpha\int_{a}^{t_{0}}h(t_{0},s) \nabla s. \end{aligned}$$

By differentiating the function \(y(t)\) and using \(\Phi ^{-1}(F^{-1}(y(t)))\leq y(t)\), we obtain \(y^{\nabla}(t)\leq\alpha f(t_{0},t) [ y(t)+\int_{a}^{t}g(t_{0},\xi)w [ y(\xi) ] \nabla\xi ]\). Let us define \(\Omega(t)=y(t)+\int_{a}^{t}g(t_{0},\xi)w [ y(\xi) ]\nabla\xi\), so \(\Omega(t)\geq y(t)\). Then

$$ \frac{\Omega^{\nabla}(t)}{\Omega(t)+w [ \Omega(t) ] }\leq\max \bigl\{ \alpha f(t_{0},t),g(t_{0},t) \bigr\} . $$
(7)

Let us define \(\hat{y}(t)\) and \(\hat{\Omega} (t)\) on \([ a,b ]_{\mathbb{T}} \) such that

$$\begin{aligned}& \hat{y}(t)=K(b)+\alpha\int_{a}^{t} \biggl( f(b,s) \biggl[ \Phi ^{-1}\bigl(z(s)\bigr)+\int_{a}^{s}g(b, \xi)w \bigl[ \Phi^{-1}\bigl(z(\xi )\bigr)\nabla \xi \bigr] \biggr] \biggr) \nabla s, \\& \hat{\Omega}(t)=\hat{y}(t)+\int_{a}^{t}g(b,\xi)w \bigl[ \hat {y}(\xi) \bigr]\nabla\xi. \end{aligned}$$

It is obvious that \(y(t)\leq\hat{y}(t)\) and \(\Omega(t)\leq\hat{\Omega }(t)\) on \([ a,t_{0} ] _{ \mathbb{T}}\). Using \(\hat{\Omega}(t)\) we define β such that

$$\beta=\max_{t\in [ a,t_{0} ] _{ \mathbb{T}}}\frac{\hat {\Omega}(t)+w(\hat{\Omega}(t))}{\Omega(a)+w(\Omega(a))}>0. $$

Again from the definition of the functions \(\Omega(t)\), \(y(t)\), \(\hat {y}(t)\), \(\hat{\Omega}(t)\), \(w(\hat{\Omega}(t))\) and by Theorem 1.65 from [12] β exists, where

$$\beta=\max_{t\in [ a,t_{0} ] _{T}}\frac{\hat{\Omega }(t)+w(\hat{\Omega}(t))}{\Omega(a)+w(\Omega(a))} \geq\frac{\Omega (t)+w(\Omega(t))}{\Omega(\rho(t))+w(\Omega(\rho(t)))}. $$

If we multiply (7) by β and use Lemma 3.2, we have

$$\Omega(t)=G^{-1} \biggl[ G\bigl(K(t_{0})\bigr)+\beta\int _{a}^{t}\max \bigl\{ \alpha f(t_{0},s),g(t_{0},s) \bigr\} \nabla s \biggr] . $$

Since \(y^{\nabla}(t)\leq\alpha f(t_{0},t)\Omega(t)\), then \(y(t)\leq K(t_{0})+\alpha\int_{a}^{t}f(t_{0},s)\Omega(s)\nabla s\). Also using the information \(u(t)\leq\Phi^{-1}(z(t))\leq\Phi ^{-1}(F^{-1}(y(t)))\), we get

$$\begin{aligned} u(t) \leq& \Phi^{-1} \biggl[ F^{-1} \biggl( K(t_{0})+\alpha \int_{a}^{t} \biggl( f(t_{0},s)G^{-1} \biggl[ G\bigl(K(t_{0})\bigr) \\ &{}+ \beta \int_{a}^{s}\max \bigl\{ \alpha f(t_{0},\xi),g(t_{0},\xi) \bigr\} \nabla\xi \biggr] \biggr) \nabla s \biggr) \biggr] . \end{aligned}$$

Since \(t_{0}\) is arbitrary, we can set \(t=t_{0}\) in the above inequality, and we get the desired result. □

Remark 3.1

The function \(G(r)\) defined above satisfies \(\lim_{r\rightarrow\infty }G(r)=\infty\) by Constantin [13]. This was discussed in [1416].

Example 3.3

If we take \(\phi(x)=\psi(x)=\Phi'(x)\) with \(\Phi'\) being nondecreasing, then \(M(x)=\Phi'(x)\), and this implies

$$F(x)=\int_{x_{0}}^{x}{\frac{1}{\Phi'\circ \Phi^{-1}(s)}}\,ds= \Phi^{-1}(x)-\Phi^{-1}(x_{0}). $$

Choose \(x_{0}=\Phi(0)\geq0\). Then \(\Phi^{-1}(\Phi(0))=0\), hence \(F(x)=\Phi^{-1}(x)\). For the particular case \(\mathbb{T}= \mathbb{Z}\), an application of Theorem 3.2 gives Theorem 3.1. For the particular case \(\mathbb{T}= \mathbb{R}\) and \(\alpha= \beta= 1\), an application of Theorem 3.2 gives the generalization of Constantin’s inequality which is done by Yang and Tan [4].

Example 3.4

Let us take \(\mathbb{T}=h \mathbb{Z}\) such that \(x,t \in[h,Mh]\), \(h(x,t)=H(x-t)\), \(g(x,t)=f(t,x)=0\), \(\Phi(x)=x^{2}\), \(\phi(x)=0\), \(\psi (x)=\frac{x}{2}\) defined for \(x\geq0\) and \(c(n)=L\), \(L\geq0 \) is constant. Then

$$M(x)=\max \biggl\{ \frac{x}{2},0 \biggr\} =\frac{x}{2}. $$

If we set \(x_{0} =0\), we obtain

$$F(x)=\int_{0}^{x}\frac{1}{\frac{\sqrt{s}}{2}}\,ds=4\sqrt{x}. $$

Thus, \(\lim_{x\rightarrow\infty}F(x)=\infty\). \(F^{-1}(x)={ (\frac {x}{4} )}^{2}\) and \(\Phi^{-1}(F^{-1}(x))= (\frac{x}{4} )\leq x\), \(\forall x\geq0\).

If we get the equality, we have

$$u^{2}(hn)=L+\sum_{t=1}^{n}hH(hn-ht) \frac{u(ht)}{2}. $$

By letting \(n=1,2\), we get

$$\begin{aligned}& u(h)=\sqrt{L+hH(0)\frac{u(h)}{2}}, \\& u(2h)=\sqrt{L+hH(h)\frac{u(h)}{2}+hH(0)\frac{u(2h)}{2}}. \end{aligned}$$

If we apply Theorem 3.2, we find an upper bound for \(u(hn)\) as

$$u(hn)\leq\sqrt{L} +\alpha\frac{\sum_{t=1}^{n}hH(hn-ht)}{4}. $$

Here

$$ \alpha=\max_{n \in[1,M]} \biggl[1+\frac{H((M-n)h)\frac {u(nh)}{2}}{L+H((M-1)h)\frac{u(h)}{2}+H((M-2)h)\frac {u(2h)}{2}+\cdots+H((M-n+1)h)\frac{u((n-1)h)}{2}} \biggr]. $$

Lemma 3.3

Let \(\mathbb{T}\) be a regulated time scale, \(a,b \in\mathbb{T}\), and consider the time scale \([a,b]_{\mathbb{T}}\) such that \(\sigma(a)=a\). Let \(r \in C^{1}([a,b]_{\mathbb{T}}, \mathbb{R})\) with \(r^{\nabla }(t),r^{\Delta}(t)\geq0\). Suppose that a function \(g\in C(\mathbb {R}_{0}^{+},\mathbb{R}_{0}^{+})\) is positive and nondecreasing on \(\mathbb{R}\).

Define \(G(x)=\int_{x_{0}}^{x} \frac{ds}{g(s)}\), where \(x\geq0\), \(x_{0} \geq 0\) if \(G(x)=\int_{0}^{x} \frac{ds}{g(s)} < \infty\) and \(x> 0\), \(x_{0} >0\) if \(G(x)=\int_{0}^{x} \frac{ds}{g(s)}=\infty\). Then, for each \(t\in[a,b]_{\mathbb{T}}\), we have

$$(G\circ r) (t)\leq2 (G\circ r) (a)+2\int_{a}^{t} \frac{r^{\diamond_{\alpha }}(s)}{g(r(\rho(s)))}. $$

Proof

By assumption we have

$$\frac{1}{g ( r(\rho(t))+h\nu(t)r^{\nabla}(t) ) } \leq \frac{1}{g ( r(\rho(t)) ) } $$

and

$$\frac{1}{g ( r(t)+h\mu(t)r^{\Delta}(t) ) } \leq \frac{1}{g ( r(t) ) }. $$

For \(\alpha\in(0,1)\), we get

$$\begin{aligned}& \alpha r^{\Delta}(t)\int_{0}^{1} \frac{dh}{g ( r (t)+h\mu(t)r^{\Delta}(t) ) } +(1-\alpha) r^{\nabla}(t)\int_{0}^{1} \frac{dh}{g ( r(\rho (t))+h\nu(t)r^{\nabla}(t) ) } \\& \quad \leq\alpha\frac{r^{\Delta}(t)}{g ( r(t) ) }+(1-\alpha) \frac {r^{\nabla}(t)}{g ( r(\rho(t)) ) } \end{aligned}$$

and

$$(G\circ r)^{\diamond_{\alpha}}(t)\leq\frac{r^{\diamond_{\alpha}}(t)}{g ( r(\rho(t)) ) }. $$

By using [8, 17] and the assumption on a, we have the desired result

$$\frac{1}{2}G\circ r(t)\leq G\circ r(a)+\int_{a}^{t} \frac{r^{\diamond_{\alpha }}(s)}{g(r(\rho(s)))}. $$

 □

Theorem 3.3

\(u\in C([a,b]_{\mathbb{T}}, \mathbb{R}_{+})\) satisfies for some \(k>0\) such that

$$u^{2}(t)\leq k^{2}+2\int_{a}^{t} \biggl[f(s) u(s) \biggl\{ u(s)+\int_{a}^{s} g(\tau )w\bigl(u(\tau)\bigr)\diamond_{\alpha}\tau \biggr\} +h(s)u(s) \biggr] \diamond_{\alpha}s. $$

Here our time scale is regulated and \(\sigma(a)=a\).

\(\forall t\in[a,b]_{\mathbb{T}}\), \(f,g,h\in C([a,b]_{\mathbb{T}}, \mathbb{R}^{0}_{+})\), \(w(t)\in C( \mathbb{R}^{0}_{+}, \mathbb{R}^{0}_{+})\) and \(w(t)\) is nondecreasing, there exist fixed constants \(m,c>0\) such that

$$\begin{aligned} u(t) \leq& 4k+2c\int_{a}^{t}\bigl(1-2\alpha+2{ \alpha}^{2}\bigr)h(\tau)+\bigl(\alpha-{\alpha }^{2}\bigr)h \bigl(\rho(\tau)\bigr)+\bigl(\alpha-{\alpha}^{2}\bigr)h\bigl(\sigma( \tau)\bigr)\diamond_{\alpha}\tau \\ &{}+2\int_{a}^{t} \bigl[\bigl(2c\bigl(1-2\alpha+2{ \alpha}^{2}\bigr)^{2}+4c\bigl(\alpha-{\alpha }^{2} \bigr)^{2}\bigr)f(s) \\ &{}+4c\bigl(1-2\alpha+2{\alpha}^{2}\bigr) \bigl(\alpha-{\alpha}^{2}\bigr) f\bigl(\rho(s)\bigr)+4c\bigl(1-2\alpha+2{\alpha}^{2}\bigr) \bigl(\alpha-{ \alpha}^{2}\bigr) f\bigl(\sigma(s)\bigr) \\ &{}+2c\bigl(\alpha -{ \alpha}^{2}\bigr)^{2}f\bigl(\sigma^{2}(s)\bigr)+2c \bigl(\alpha-{\alpha}^{2}\bigr)^{2}f\bigl(\rho ^{2}(s)\bigr) \bigr] \\ &{}\times \biggl[E^{-1} \biggl\{ 2E \biggl(2k+c\int _{a}^{t_{0}}\bigl(1-2\alpha+2{\alpha }^{2} \bigr)h(\tau)+\bigl(\alpha-{\alpha}^{2}\bigr)h\bigl(\rho(\tau)\bigr) \\ &{}+ \bigl(\alpha-{\alpha}^{2}\bigr)h\bigl(\sigma (\tau)\bigr) \diamond_{\alpha}\tau \biggr)+\int_{a}^{s} \bigl[\bigl(4mc\bigl(1-2 \alpha+2{\alpha}^{2}\bigr)^{2}+8mc\bigl(\alpha-{\alpha }^{2}\bigr)^{2}\bigr)f\bigl(\sigma^{2}(\tau) \bigr) \\ &{}+8mc\bigl(1-2\alpha+2{\alpha}^{2}\bigr) \bigl(\alpha-{\alpha }^{2}\bigr) f\bigl(\sigma(\tau)\bigr)+8mc\bigl(1-2\alpha+2{\alpha}^{2}\bigr) \bigl(\alpha-{ \alpha}^{2}\bigr) f\bigl(\sigma^{3}(\tau )\bigr) \\ &{}+4mc\bigl( \alpha-{\alpha}^{2}\bigr)^{2}f\bigl(\sigma^{4}( \tau)\bigr)+4mc\bigl(\alpha-{\alpha }^{2}\bigr)^{2}f(\tau) \bigr]\diamond_{\alpha}\tau \\ &{}+ \int_{a}^{s} \bigl[2\bigl(1-2\alpha+2{ \alpha}^{2}\bigr)m g\bigl(\sigma^{2}(\tau )\bigr)+2\bigl( \alpha-{\alpha}^{2}\bigr)m g\bigl(\sigma(\tau)\bigr) \\ &{}+2\bigl(\alpha-{ \alpha}^{2}\bigr)m g\bigl(\sigma^{3}(\tau)\bigr) \bigr] \diamond_{\alpha}\tau \biggr\} \biggr]\diamond _{\alpha}s, \end{aligned}$$

where \(E(r)=\int_{1}^{r} \frac{ds}{w(s)+s}\), \(r>0\).

Proof

If we take \(a=t\), then the inequality obviously holds true. Let \(t_{0}\in (a,b]\) and define \(z(t)\) in \([a,t_{0}]\) such that

$$z(t)=k^{2}+2\int_{a}^{t} \biggl[f(s) u(s) \biggl\{ u(s)+\int_{a}^{s} g(\tau)w\bigl(u( \tau )\bigr)\diamond_{\alpha}\tau \biggr\} +h(s)u(s) \biggr] \diamond_{\alpha}s. $$

Therefore, for \(u(t)\leq\sqrt{z(t)}\), \(t\in[a,t_{0}]_{\mathbb{T}}\). Since \(z^{\Delta}(t), z^{\nabla}(t)\geq0\), if we use [8], we obtain

$$\begin{aligned} 0 \leq& z^{\diamond_{\alpha}}(t) \\ \leq&2\bigl(1-2\alpha+2{\alpha}^{2}\bigr) \biggl[f(t) \sqrt{z(t)} \biggl\{ \sqrt{z(t)}+\int_{a}^{t} g(\tau)w\bigl( \sqrt{z(\tau )}\bigr)\diamond_{\alpha}\tau \biggr\} +h(t)\sqrt{z(t)} \biggr] \\ &{}+2\bigl(\alpha-{\alpha}^{2}\bigr) \biggl[f\bigl(\rho(t)\bigr) \sqrt{z\bigl(\rho(t)\bigr)} \biggl\{ \sqrt {z\bigl(\rho(t)\bigr)} +\int _{a}^{\rho(t)} g(\tau)w\bigl( \sqrt{z(\tau)}\bigr) \diamond_{\alpha}\tau \biggr\} \\ &{}+h\bigl(\rho(t)\bigr) \sqrt{z\bigl(\rho(t) \bigr)} \biggr] \\ &{}+2\bigl(\alpha-{\alpha}^{2}\bigr) \biggl[f\bigl(\sigma(t)\bigr) \sqrt{z\bigl(\sigma(t)\bigr)} \biggl\{ \sqrt{z\bigl(\sigma(t)\bigr)}+\int _{a}^{\sigma(t)} g(\tau)w\bigl( \sqrt{z(\tau)}\bigr) \diamond _{\alpha}\tau \biggr\} \\ &{}+h\bigl(\sigma(t)\bigr)\sqrt{z\bigl(\sigma(t) \bigr)} \biggr]. \end{aligned}$$

Then \(z(t)\) is nondecreasing. In other words, \(z(\sigma(t))\geq z(t) \geq z(\rho(t))\), so we have

$$\begin{aligned} \frac{z^{\diamond_{\alpha}}(t)}{2\sqrt{z(\sigma(t))} } \leq&\bigl(1-2\alpha +2{\alpha}^{2} \bigr) \biggl[f(t) \biggl\{ \sqrt{z(t)}+\int_{a}^{t} g(\tau)w\bigl( \sqrt {z(\tau)}\bigr)\diamond_{\alpha}\tau \biggr\} +h(t) \biggr] \\ &{}+\bigl(\alpha-{\alpha}^{2}\bigr) \biggl[f\bigl(\rho(t)\bigr) \biggl\{ \sqrt{z\bigl(\rho(t)\bigr)} +\int_{a}^{\rho(t)} g(\tau)w\bigl( \sqrt{z(\tau)}\bigr)\diamond_{\alpha}\tau \biggr\} \\ &{}+h\bigl( \rho (t)\bigr) \biggr] \\ &{}+\bigl(\alpha-{\alpha}^{2}\bigr) \biggl[f\bigl(\sigma(t)\bigr) \biggl\{ \sqrt{z\bigl(\sigma(t)\bigr)}+\int_{a}^{\sigma(t)} g(\tau)w\bigl( \sqrt{z(\tau)}\bigr)\diamond_{\alpha}\tau \biggr\} \\ &{}+h\bigl( \sigma(t)\bigr) \biggr]. \end{aligned}$$
(8)

Functions that compose \(z(t)\) are from \(C([a,b]_{\mathbb{T}}, \mathbb {R}^{0}_{+})\), then they are regulated on \([a,b]_{\mathbb{T}}\), then \(z(t)\) is bounded and \(z(t)\) never takes zero. Therefore there exists c such that \(c=\max_{t\in[a,t_{0}]_{\mathbb{T}}}\frac{\sqrt{z(\sigma(t))}}{\sqrt {z(\rho(t))}}\).

Multiply (8) by c and obtain

$$\begin{aligned} \frac{z^{\diamond_{\alpha}}(t)}{2\sqrt{z(\rho(t))} } \leq& c\bigl(1-2\alpha +2{\alpha}^{2}\bigr) \biggl[f(t) \biggl\{ \sqrt{z(t)}+\int_{a}^{t} g( \tau)w\bigl( \sqrt {z(\tau)}\bigr)\diamond_{\alpha}\tau \biggr\} +h(t) \biggr] \\ &{}+c\bigl(\alpha-{\alpha}^{2}\bigr) \biggl[f\bigl(\rho(t)\bigr) \biggl\{ \sqrt{z\bigl(\rho(t)\bigr)} +\int_{a}^{\rho(t)} g(\tau)w\bigl( \sqrt{z(\tau)}\bigr)\diamond_{\alpha}\tau \biggr\} +h\bigl( \rho (t)\bigr) \biggr] \\ &{}+c\bigl(\alpha-{\alpha}^{2}\bigr) \biggl[f\bigl(\sigma(t)\bigr) \biggl\{ \sqrt{z\bigl(\sigma (t)\bigr)}+\int_{a}^{\sigma(t)} g(\tau)w\bigl( \sqrt{z(\tau)}\bigr)\diamond_{\alpha}\tau \biggr\} +h\bigl( \sigma(t)\bigr) \biggr]. \end{aligned}$$

Now we use Lemma 3.3 and we get

$$\begin{aligned} \sqrt{z(t)} \leq&2k+2c\bigl(1-2\alpha+2{\alpha}^{2}\bigr)\int _{a}^{t} \biggl[f(s) \biggl\{ \sqrt{z(s)}+\int _{a}^{s} g(\tau)w\bigl( \sqrt{z(\tau)}\bigr) \diamond_{\alpha}\tau \biggr\} \biggr]\diamond_{\alpha}s \\ &{}+2c\bigl(\alpha-{\alpha}^{2}\bigr)\int_{a}^{t} \biggl[f\bigl(\rho(s)\bigr) \biggl\{ \sqrt{z\bigl(\rho (s)\bigr)} +\int _{a}^{\rho(s)} g(\tau)w\bigl( \sqrt{z(\tau)}\bigr) \diamond_{\alpha}\tau \biggr\} \biggr]\diamond_{\alpha}s \\ &{}+2c\bigl(\alpha-{\alpha}^{2}\bigr)\int_{a}^{t} \biggl[f\bigl(\sigma(s)\bigr) \biggl\{ \sqrt {z\bigl(\sigma(s)\bigr)}+\int _{a}^{\sigma(s)} g(\tau)w\bigl( \sqrt{z(\tau)}\bigr) \diamond _{\alpha}\tau \biggr\} \biggr]\diamond_{\alpha}s \\ &{}+\int_{a}^{t_{0}}\bigl(1-2\alpha+2{ \alpha}^{2}\bigr)h(s)+\bigl(\alpha-{\alpha}^{2}\bigr)h\bigl( \rho (s)\bigr)+\bigl(\alpha-{\alpha}^{2}\bigr)h\bigl(\sigma(s)\bigr) \diamond_{\alpha}s. \end{aligned}$$

Let us say the right-hand side of the above inequality is \(V(t)\), then \(\sqrt{z(t)}\leq V(t)\). So we get

$$\begin{aligned} V^{\diamond_{\alpha}}(t) \leq&\bigl(2c\bigl(1-2\alpha+2{\alpha}^{2} \bigr)^{2}+4c\bigl(\alpha -{\alpha}^{2}\bigr)^{2} \bigr) \biggl[f(t) \biggl\{ V(t)+\int_{a}^{t} g( \tau)w\bigl(V(\tau )\bigr)\diamond_{\alpha}\tau \biggr\} \biggr] \\ &{}+4c\bigl(1-2\alpha+2{\alpha}^{2}\bigr) \bigl(\alpha-{ \alpha}^{2}\bigr) \biggl[f\bigl(\rho(t)\bigr) \biggl\{ V\bigl(\rho(t) \bigr) +\int_{a}^{\rho(s)} g(\tau)w\bigl( V(\tau)\bigr) \diamond_{\alpha}\tau \biggr\} \biggr] \\ &{}+4c\bigl(1-2\alpha+2{\alpha}^{2}\bigr) \bigl(\alpha-{ \alpha}^{2}\bigr) \biggl[f\bigl(\sigma(t)\bigr) \biggl\{ V\bigl(\sigma(t) \bigr)+\int_{a}^{\sigma(t)} g(\tau)w\bigl( V(\tau)\bigr) \diamond_{\alpha}\tau \biggr\} \biggr] \\ &{}+ 2c\bigl(\alpha-{\alpha}^{2}\bigr)^{2} \biggl[f\bigl( \rho^{2}(t)\bigr) \biggl\{ V\bigl(\rho^{2}(t)\bigr) +\int _{a}^{\rho^{2}(t)} g(\tau)w\bigl( V(\tau)\bigr) \diamond_{\alpha}\tau \biggr\} \biggr] \\ &{}+ 2c\bigl(\alpha-{\alpha}^{2}\bigr)^{2} \biggl[f\bigl( \sigma^{2}(t)\bigr) \biggl\{ V\bigl(\sigma ^{2}(t)\bigr) + \int_{a}^{\sigma^{2}(t)} g(\tau)w\bigl( V(\tau)\bigr) \diamond_{\alpha}\tau \biggr\} \biggr]. \end{aligned}$$

Here \(V^{\diamond_{\alpha}}(t)\geq0\). Similarly, if we take the delta and nabla derivative of \(V(t)\), we also see that \(V^{\Delta}(t), V^{\nabla}(t)\geq0\).

Therefore \(V(\sigma^{2}(t))\geq V(\sigma(t))\geq V(t)\geq V(\rho (t))\geq V(\rho^{2}(t))\). Then

$$\begin{aligned} V^{\diamond_{\alpha}}(t) \leq& \bigl[\bigl(2c\bigl(1-2\alpha+2{ \alpha}^{2}\bigr)^{2}+4c\bigl(\alpha -{\alpha}^{2} \bigr)^{2}\bigr)f(t)+4c\bigl(1-2\alpha+2{\alpha}^{2}\bigr) \bigl(\alpha-{\alpha}^{2}\bigr) f\bigl(\rho(t)\bigr) \\ &{}+4c\bigl(1-2\alpha+2{\alpha}^{2}\bigr) \bigl(\alpha-{ \alpha}^{2}\bigr) f\bigl(\sigma(t)\bigr)+2c\bigl(\alpha -{ \alpha}^{2}\bigr)^{2}f\bigl(\sigma^{2}(t)\bigr) \\ &{}+2c \bigl(\alpha-{\alpha}^{2}\bigr)^{2}f\bigl(\rho ^{2}(t)\bigr) \bigr] \biggl\{ V\bigl(\sigma^{2}(t)\bigr) +\int _{a}^{\sigma^{2}(t)} g(\tau)w\bigl( V(\tau )\bigr) \diamond_{\alpha}\tau \biggr\} . \end{aligned}$$
(9)

Let us take \(V(\sigma^{2}(t)) +\int_{a}^{\sigma^{2}(t)} g(\tau)w( V(\tau ))\diamond_{\alpha}\tau=\Omega(t)\). Taking the nabla derivative of \(\Omega (t)\) and using (9), we have

$$\begin{aligned} \Omega^{\diamond_{\alpha}}(t) \leq& \bigl[\bigl(2c\bigl(1-2\alpha+2{\alpha }^{2}\bigr)^{2}+4c\bigl(\alpha-{\alpha}^{2} \bigr)^{2}\bigr)f\bigl(\sigma^{2}(t)\bigr) \\ &{}+4c\bigl(1-2 \alpha+2{\alpha }^{2}\bigr) \bigl(\alpha-{\alpha}^{2}\bigr) f \bigl(\sigma(t)\bigr) \\ &{}+4c\bigl(1-2\alpha+2{\alpha}^{2}\bigr) \bigl(\alpha-{ \alpha}^{2}\bigr) f\bigl(\sigma ^{3}(t)\bigr) \\ &{}+2c\bigl( \alpha-{\alpha}^{2}\bigr)^{2}f\bigl(\sigma^{4}(t) \bigr)+2c\bigl(\alpha-{\alpha }^{2}\bigr)^{2}f(t) \bigr] \Omega(t) \\ &{}+\bigl(1-2\alpha+2{\alpha}^{2}\bigr)g\bigl(\sigma^{2}(t) \bigr)w\bigl( V\bigl(\sigma^{2}(t)\bigr)\bigr)+\bigl(\alpha -{ \alpha}^{2}\bigr)g\bigl(\sigma(t)\bigr)w\bigl( V\bigl(\sigma(t)\bigr) \bigr) \\ &{}+\bigl(\alpha-{\alpha}^{2}\bigr)g\bigl(\sigma^{3}(t) \bigr)w\bigl( V\bigl(\sigma^{3}(t)\bigr)\bigr), \end{aligned}$$

\(\Omega^{\diamond_{\alpha}}(t)\geq0\). Similarly, \(\Omega^{\Delta}(t)\), \(\Omega^{\nabla}(t)\geq0\). It is obvious that \(V(\sigma^{2}(t))\leq \Omega(t)\)and since w is nondecreasing, we also have \(w(\Omega(\sigma (t)))\geq w(\Omega(t))\geq w(\Omega(\rho(t)))\). Therefore \(w(\Omega (\sigma(t)))\geq w( V(\sigma^{3}(t)))\),

$$\begin{aligned}& \frac{\Omega^{\diamond_{\alpha}}(t)}{\Omega(t)+w(\Omega(\sigma(t)))} \\& \quad \leq \bigl[\bigl(2c\bigl(1-2\alpha+2{ \alpha}^{2}\bigr)^{2}+4c\bigl(\alpha-{\alpha}^{2} \bigr)^{2}\bigr)f\bigl(\sigma ^{2}(t)\bigr)+4c\bigl(1-2 \alpha+2{\alpha}^{2}\bigr) \bigl(\alpha-{\alpha}^{2}\bigr) f \bigl(\sigma(t)\bigr) \\& \qquad {}+4c\bigl(1-2\alpha+2{\alpha}^{2}\bigr) \bigl(\alpha-{ \alpha}^{2}\bigr) f\bigl(\sigma ^{3}(t)\bigr)+2c\bigl( \alpha-{\alpha}^{2}\bigr)^{2}f\bigl(\sigma^{4}(t) \bigr)+2c\bigl(\alpha-{\alpha }^{2}\bigr)^{2}f(t) \bigr] \\& \qquad {}+ \bigl[\bigl(1-2\alpha+2{\alpha}^{2}\bigr)g\bigl( \sigma^{2}(t)\bigr)+\bigl(\alpha-{\alpha }^{2}\bigr)g\bigl( \sigma(t)\bigr)+\bigl(\alpha-{\alpha}^{2}\bigr)g\bigl( \sigma^{3}(t)\bigr) \bigr]. \end{aligned}$$
(10)

By using the same information while finding the existence of c, there exists m such that \(m=\max_{t\in[a,t_{0}]_{\mathbb{T}}}\frac{\Omega (t)+w(\Omega(\sigma(t)))}{\Omega(\rho(t))+w(\Omega(\rho(t)))}\). If we multiply (10) by m, we have

$$\begin{aligned}& \frac{\Omega^{\diamond_{\alpha}}(t)}{\Omega(\rho(t))+w(\Omega(\rho (t)))} \\& \quad \leq \bigl[\bigl(2mc\bigl(1-2\alpha+2{\alpha}^{2} \bigr)^{2}+4mc\bigl(\alpha-{\alpha }^{2}\bigr)^{2} \bigr)f\bigl(\sigma^{2}(t)\bigr) \\& \qquad {}+4mc\bigl(1-2\alpha+2{\alpha}^{2}\bigr) \bigl(\alpha-{ \alpha}^{2}\bigr) f\bigl(\sigma(t)\bigr) \\& \qquad {}+4mc\bigl(1-2\alpha+2{\alpha}^{2}\bigr) \bigl(\alpha-{ \alpha}^{2}\bigr) f\bigl(\sigma ^{3}(t)\bigr) \\& \qquad {}+2mc\bigl( \alpha-{\alpha}^{2}\bigr)^{2}f\bigl(\sigma^{4}(t) \bigr)+2mc\bigl(\alpha-{\alpha }^{2}\bigr)^{2}f(t) \bigr] \\& \qquad {}+ \bigl[\bigl(1-2\alpha+2{\alpha}^{2}\bigr)m g\bigl( \sigma^{2}(t)\bigr)+\bigl(\alpha-{\alpha }^{2}\bigr)m g\bigl( \sigma(t)\bigr)+\bigl(\alpha-{\alpha}^{2}\bigr)m g\bigl( \sigma^{3}(t)\bigr) \bigr]. \end{aligned}$$

Now we use Lemma 3.3 and get

$$\begin{aligned} \Omega(t) \leq& E^{-1} \biggl\{ 2E \biggl(2k+c\int_{a}^{t_{0}} \bigl(1-2\alpha+2{\alpha }^{2}\bigr)h(s) \\ &{}+\bigl(\alpha-{ \alpha}^{2}\bigr)h\bigl(\rho(s)\bigr)+\bigl(\alpha-{ \alpha}^{2}\bigr)h\bigl(\sigma (s)\bigr)\diamond_{\alpha}s \biggr) \\ &{}+\int_{a}^{t} \bigl[\bigl(4mc\bigl(1-2 \alpha+2{\alpha}^{2}\bigr)^{2}+8mc\bigl(\alpha-{\alpha }^{2}\bigr)^{2}\bigr)f\bigl(\sigma^{2}(s)\bigr) \\ &{}+8mc\bigl(1-2\alpha+2{\alpha}^{2}\bigr) \bigl(\alpha-{ \alpha}^{2}\bigr) f\bigl(\sigma (s)\bigr)+8mc\bigl(1-2\alpha+2{ \alpha}^{2}\bigr) \bigl(\alpha-{\alpha}^{2}\bigr) f\bigl( \sigma^{3}(s)\bigr) \\ &{}+4mc\bigl(\alpha-{\alpha}^{2}\bigr)^{2}f\bigl( \sigma^{4}(s)\bigr)+4mc\bigl(\alpha-{\alpha }^{2} \bigr)^{2}f(s) \bigr]\diamond_{\alpha}s \\ &{}+ \int_{a}^{t} \bigl[2\bigl(1-2\alpha+2{ \alpha}^{2}\bigr)m g\bigl(\sigma^{2}(s)\bigr)+2\bigl(\alpha -{\alpha}^{2}\bigr)m g\bigl(\sigma(s)\bigr) \\ &{}+2\bigl(\alpha-{ \alpha}^{2}\bigr)m g\bigl(\sigma ^{3}(s)\bigr) \bigr] \diamond_{\alpha}s \biggr\} . \end{aligned}$$

Then we have

$$\begin{aligned} V^{\diamond_{\alpha}}(t) \leq& \bigl[\bigl(2c\bigl(1-2\alpha+2{ \alpha}^{2}\bigr)^{2}+4c\bigl(\alpha -{\alpha}^{2} \bigr)^{2}\bigr)f(t)+4c\bigl(1-2\alpha+2{\alpha}^{2}\bigr) \bigl(\alpha-{\alpha}^{2}\bigr) f\bigl(\rho(t)\bigr) \\ &{}+4c\bigl(1-2\alpha+2{\alpha}^{2}\bigr) \bigl(\alpha-{ \alpha}^{2}\bigr) f\bigl(\sigma(t)\bigr)+2c\bigl(\alpha -{ \alpha}^{2}\bigr)^{2}f\bigl(\sigma^{2}(t)\bigr) \\ &{}+2c \bigl(\alpha-{\alpha}^{2}\bigr)^{2}f\bigl(\rho ^{2}(t)\bigr) \bigr] E^{-1} \biggl\{ 2E \biggl(2k+c\int_{a}^{t_{0}} \bigl(1-2\alpha+2{\alpha }^{2}\bigr)h(s) \\ &{}+\bigl(\alpha-{ \alpha}^{2}\bigr)h\bigl(\rho(s)\bigr)+\bigl(\alpha-{ \alpha}^{2}\bigr)h\bigl(\sigma (s)\bigr)\diamond_{\alpha}s \biggr) \\ &{}+\int_{a}^{t} \bigl[\bigl(4mc\bigl(1-2 \alpha+2{\alpha}^{2}\bigr)^{2}+8mc\bigl(\alpha-{\alpha }^{2}\bigr)^{2}\bigr)f\bigl(\sigma^{2}(s)\bigr) \\ &{}+8mc\bigl(1-2\alpha+2{\alpha}^{2}\bigr) \bigl(\alpha-{ \alpha}^{2}\bigr) f\bigl(\sigma (s)\bigr)+8mc\bigl(1-2\alpha+2{ \alpha}^{2}\bigr) \bigl(\alpha-{\alpha}^{2}\bigr) f\bigl( \sigma^{3}(s)\bigr) \\ &{}+4mc\bigl(\alpha-{\alpha}^{2}\bigr)^{2}f\bigl( \sigma^{4}(s)\bigr)+4mc\bigl(\alpha-{\alpha }^{2} \bigr)^{2}f(s) \bigr]\diamond_{\alpha}s \\ &{}+\int_{a}^{t} \bigl[2\bigl(1-2\alpha+2{ \alpha}^{2}\bigr)m g\bigl(\sigma^{2}(s)\bigr)+2\bigl(\alpha -{\alpha}^{2}\bigr)m g\bigl(\sigma(s)\bigr) \\ &{}+2\bigl(\alpha-{ \alpha}^{2}\bigr)m g\bigl(\sigma ^{3}(s)\bigr) \bigr] \diamond_{\alpha}s \biggr\} . \end{aligned}$$
(11)

Using [8] we obtain

$$\begin{aligned} \frac{1}{2}V(t) \leq&2k+c\int_{a}^{t_{0}} \bigl(1-2\alpha+2{\alpha}^{2}\bigr)h(\tau )+\bigl(\alpha-{ \alpha}^{2}\bigr)h\bigl(\rho(\tau)\bigr)+\bigl(\alpha-{ \alpha}^{2}\bigr)h\bigl(\sigma(\tau )\bigr)\diamond_{\alpha}\tau \\ &{}+\int_{a}^{t} \bigl[\bigl(2c\bigl(1-2 \alpha+2{\alpha}^{2}\bigr)^{2}+4c\bigl(\alpha-{\alpha }^{2}\bigr)^{2}\bigr)f(s) \\ &{}+4c\bigl(1-2\alpha+2{ \alpha}^{2}\bigr) \bigl(\alpha-{\alpha}^{2}\bigr) f\bigl( \rho(s)\bigr) +4c\bigl(1-2\alpha+2{\alpha}^{2}\bigr) \bigl(\alpha-{ \alpha}^{2}\bigr) f\bigl(\sigma(s)\bigr) \\ &{}+2c\bigl(\alpha -{ \alpha}^{2}\bigr)^{2}f\bigl(\sigma^{2}(s)\bigr)+2c \bigl(\alpha-{\alpha}^{2}\bigr)^{2}f\bigl(\rho ^{2}(s)\bigr) \bigr] \\ &{}\times \biggl[E^{-1} \biggl\{ 2E \biggl(2k+c\int _{a}^{t_{0}}\bigl(1-2\alpha+2{\alpha }^{2} \bigr)h(\tau) \\ &{}+\bigl(\alpha-{\alpha}^{2}\bigr)h\bigl(\rho(\tau)\bigr)+ \bigl(\alpha-{\alpha}^{2}\bigr)h\bigl(\sigma (\tau)\bigr) \diamond_{\alpha}\tau \biggr) \\ &{}+\int_{a}^{s} \bigl[\bigl(4mc\bigl(1-2 \alpha+2{\alpha}^{2}\bigr)^{2}+8mc\bigl(\alpha-{\alpha }^{2}\bigr)^{2}\bigr)f\bigl(\sigma^{2}(\tau) \bigr) \\ &{}+8mc\bigl(1-2\alpha+2{\alpha}^{2}\bigr) \bigl(\alpha-{\alpha }^{2}\bigr) f\bigl(\sigma(\tau)\bigr)+8mc\bigl(1-2\alpha+2{\alpha}^{2}\bigr) \bigl(\alpha-{ \alpha}^{2}\bigr) f\bigl(\sigma^{3}(\tau )\bigr) \\ &{}+4mc\bigl( \alpha-{\alpha}^{2}\bigr)^{2}f\bigl(\sigma^{4}( \tau)\bigr)+4mc\bigl(\alpha-{\alpha }^{2}\bigr)^{2}f(\tau) \bigr]\diamond_{\alpha}\tau \\ &{}+ \int_{a}^{s} \bigl[2\bigl(1-2\alpha+2{ \alpha}^{2}\bigr)m g\bigl(\sigma^{2}(\tau )\bigr)+2\bigl( \alpha-{\alpha}^{2}\bigr)m g\bigl(\sigma(\tau)\bigr) \\ &{}+2\bigl(\alpha-{ \alpha}^{2}\bigr)m g\bigl(\sigma^{3}(\tau)\bigr) \bigr] \diamond_{\alpha}\tau \biggr\} \biggr]\diamond _{\alpha}s. \end{aligned}$$

Since \(u(t)\leq\sqrt{z(t)}\leq V(t)\), we get the desired result. □

Example 3.5

If we choose \(\alpha=1\), \(\mathbb{T}=\mathbb{R}\), \(m=c=1/2\), then Theorem 3.3 becomes Theorem 1.1.

Example 3.6

Let \(\mathbb{T}=\{0\}\cup\{\frac{1}{n+1}:n\in\mathbb{N}\}\cup \mathbb{N}\). Here we assume that \(\mathbb{N}\) starts with 1. It is obvious that our time scales is regulated and \(0=\sigma(0)\). Let us choose \(M\in\mathbb{N}\) and investigate the following equality on the time scales \([0,M]_{\mathbb{T}}\):

$$u^{2}(t)=k^{2}+2\int_{0}^{t} h(s)u(s) \diamond_{\alpha}s, $$

\(h(t)=0\) if \(t\in\{0\}\cup\{\frac{1}{n+1}:n\in\mathbb{N}\}\) and \(h(t)=P(t)\) if \(t\in \mathbb{N}\). Therefore \(u(t)=k\) if \(t\in\{0\} \cup\{\frac{1}{n+1}:n\in\mathbb{N}\}\), \(u^{2}(n)=k^{2}+P(1)u(1)+P(2)u(2)+\cdots+P(n-1)u(n-1)+(1-\alpha)P(n)u(n)\) if \(n \in[0,M]\). If we apply Theorem 3.3, we get the bound for \(u(t)\) such that

$$\begin{aligned} u(t) \leq&4k+2c\sum_{\tau=1}^{t-1} \bigl(1-2 \alpha+2{\alpha}^{2}\bigr)P(\tau)+\bigl(\alpha -{\alpha}^{2} \bigr)P(\tau-1)+\bigl(\alpha-{\alpha}^{2}\bigr)P(\tau+1) \\ &{}+ 2c(1-\alpha) \bigl[\bigl(1-2\alpha+2{\alpha}^{2}\bigr)P(t)+ \bigl(\alpha-{\alpha }^{2}\bigr)P(t-1)+\bigl(\alpha-{ \alpha}^{2}\bigr)P(t+1) \bigr], \end{aligned}$$

where

$$ c=\max_{n\in[1,M]}\sqrt{1+\frac{2P(n-1)u(n-1)+2(1-\alpha )P(n)u(n)}{k^{2}+2P(1)u(1)+2P(2)u(2)+\cdots+2P(n-2)u(n-2)}}. $$

If we take \(\alpha=\frac{1}{2}\), \(k^{2}=2\), then for the equation \(u^{2}(t)=2+2\int_{0}^{t} h(s)u(s) \diamond_{\alpha}s\), where \(h(s)=0\), if \(t\in\{0\}\cup\{\frac{1}{n+1}:n\in\mathbb{N}\}\) and \(h(s)=1\) if \(t\in\mathbb{N}\), then \(c=\frac{1}{2}\) and \(u(t)\) is bound with \(u(t)\leq\sqrt{2}\) if \(t\in\{0\}\cup\{\frac{1}{n+1}:n\in\mathbb {N}\}\) and \(u(n)\leq4\sqrt{2}+n-\frac{1}{2} \) if \(n\in\mathbb{N}\).